0 Daumen
1k Aufrufe

Ich habe hier zwei Aufgaben die ich nicht machen kann, bzw. nicht verstehe....

Ich verstehe einfach das Thema nicht. Kann mir vielleicht jemand helfen? Aber auch Schrittweise versuchen zu erklären? Ich würde mich sehr freuen, wenn mir es jemand erklärt, sodass ich es auch verstehe.

Aufgabe 1) Lösen Sie die Gleichung, wobei A eine reelle 2*4 Matrix und  $$\overrightarrow { x } \in { C }^{ 2 }$$ ist.

a)   $$4\left( \begin{matrix} 1 & 2 & 3 \\ 5 & 6 & 7 \end{matrix}\begin{matrix} 4 \\ 8 \end{matrix} \right) -3A=\left( \begin{matrix} \begin{matrix} -1 \\ 0 \end{matrix} & \begin{matrix} 4 \\ 4 \end{matrix} \\ -1 & 6 \\ -1 & 8 \end{matrix} \right)$$

b) $$\begin{pmatrix} 1 & 2 \\ 5 & 6 \end{pmatrix}*\xrightarrow { X } = \left( \begin{matrix} i \\ -2 \end{matrix} \right)$$


1.1) Man betrachte das folgende Lineare Gleichungssystem

$$\begin{matrix} ax+y+6z & = & 9 \\ x+y+3z & = & 4 \\ x-2y-3z & = & -4 \end{matrix}$$


in den Unbekannten x,y und z. Dabei ist a eine reelle Konstante. Man berechne mit Hilfe des Gauß Algorithmus die konstante a der art,  dass dieses Gleichungssystem keine Lösung besitzt!

Avatar von

bei 1b bitte einmal korrigieren, formeleditor funktionert nicht, ich kann die matrix nicht zeigen lasse!

$$ \text{1.b)}\quad \begin{pmatrix} 1 & 2 \\ 5 & 6 \end{pmatrix} \cdot \overrightarrow{ x } = \begin{pmatrix} \text{i} \\ -2 \end{pmatrix} $$

So vielleicht?
Dann wäre die Multiplikation der Gleichung von links mit der Inversen der Ansatz:
$$ \text{1.b)}\quad \begin{pmatrix} -\frac { 3 }{ 2 } & \frac { 1 }{ 2 } \\ \frac { 5 }{ 4 } & -\frac { 1 }{ 4 } \end{pmatrix} \cdot \begin{pmatrix} 1 & 2 \\ 5 & 6 \end{pmatrix} \cdot \overrightarrow{ x } = \begin{pmatrix} -\frac { 3 }{ 2 } & \frac { 1 }{ 2 } \\ \frac { 5 }{ 4 } & -\frac { 1 }{ 4 } \end{pmatrix} \cdot \begin{pmatrix} \text{i} \\ -2 \end{pmatrix} $$...

jaa genau so soll es sein....gott sei dank endlich mal jemand der mich versteht.....vielen dank für deine korrektur!


bei dieser aufgabe komme ich nicht weiter :(

wie kommst du jetzt auf die  $$-\frac { 3 }{ 2 } ,\frac { 1 }{ 2 } ,\frac { 5 }{ 4 } \quad usw$$


was hast du den multipliziert :(((  ich komme nicht weiter  :(((

was muss man jetzt rechnen? Wir haben ja noch "i" in der Rechnung

Ich habe die zu der Matrix vor dem Vektor x inverse Matrix berechnet. Dann habe ich die beiden Seiten der Gleichung jeweils von links mit dieser inversen Matrix multipliziert. Diese Umformung habe ich aber noch nicht ausgerechnet, sondern nur notiert, was ich machen möchte. Im nächsten Schritt werden die beiden Seiten berechnet, wobei links die zweireihige Einheitsmatrix entsteht, die dann weggelassen wird, so dass unser Ziel, die Gleichung nach dem Vektor x aufzulösen, erreicht ist. Auch die rechte Seite muss ausgerechnet werden, es entsteht ein komplexer Vektor, die Lösung!
Könntest du das vielleicht nochmal bildlich darstellen? Ich habe nur die hälfte verstanden....du hast also quasi erstmal die erste matrix mit x mal genommen, richtig? daher kommen die werte 3/2, 1/2 usw....
und dann??
ich hatte noch nie solche großen probleme in mathe gehabt, war zwar nie die beste aber zurzeit was das thema matrizen betrifft habe ich abgeschaltet, ich kann es irgendwie nicht ganz nachvollziehen
Du solltest die folgenden Fragen zum Thema "Rechnen mit Matrizen" beantworten können:

- Was ist eine Matrix?
- Was ist der Typ einer Matrix?
- Wie wird eine Matrix transponiert?
- Wie werden Matrizen addiert bzw. skalarmultipliziert (vervielfacht)?
- Wie werden Matrizen miteinander multipliziert und wie müssen diese beschaffen sein?
- Was ist die Inverse einer Matrix und wie lässt sich diese bestimmen?

Deine Aufgaben setzen das bereits als bekannt voraus. Schau also mal ins Buch / Skript, ob da was zum Rechnen mit Matrizen steht. Anwendungsbeispiele kannst Du einstweilen überspringen.

1 Antwort

+1 Daumen
Irgendwas ist da falsch bei a)
links steht eine Matrix mit 2 Zeilen und rechts mit 4
war da vielleicht irgendwo was mit  "transponiert"   ( wie hoch t ) geschrieben.
Avatar von 288 k 🚀

Genau da kommt noch das hoch "t" dazu...ich wusste bloß nicht wie ich das machen sollte

dann sieht das os aus
Das hoch t heißt : die erste Zeile wird zur 1. Spalte etc.

4 *   1   2    3     4      -  3   A      =     -1   0   -1      -1
        5    6   7     8                              4   4   6       8

        4   8    12    16      -  3   A      =     -1   0   -1      -1
      20   24   28    32                              4   4   6       8

Jetzt die linke Matrix von der rechten abziehen gibt

                                    -  3   A      =       -5   -8   -11      -15
                                                          -16   -20   -22       -24

Jetzt alle Matrixelemente durch -3 teilen, dann hast du das A.
vielen dank für deine Hilfe....ich hätte gar nicht gedacht, dass das quasi nur zum Schluss division ist. 
Beim Formel editor habe ich ein problem, irgendwie kann ich meine Matrix nicht aufstellen. Könntest du mir auch bei b) dann quasi helfen. ich versuch das mal aufzustellen....
bei b) hat man eine zweier matrix.                                                                       1 2 5 6   das wird dann mal genommen, * mit X( pfeil über dem X) =    i                                                                                                                      -2    ( soll ein zweier matrix sein, einfach i und unten -2)  




und über dem "x" steht ein Pfeil wie bei dem Thema Geraden....
$$\begin{pmatrix} 1 & 2 \\ 5 & 6 \end{pmatrix}*\overrightarrow { X } =\left( \begin{matrix} i \\ -2 \end{matrix} \right)$$

ich verstehe es einfach nicht, die erste Matrix konnte ich noch aufstellen, dass aber funktioniert nicht :(( Oben habe ich es versucht aufzustellen, ich hoffe Ihr versteht was ich meine....
die erste Aufgabe, sprich Teil a) habe ich verstanden, aber die anderen nicht. Wie löse ich denn die?
Ps: spielt das "t" eine große Rolle bei Matrizen? Was sagt mir das denn aus?
Bild Mathematik das ist aufgaben Teil b)  Ich weiß gar nicht warum der Admin das nicht korrigiert....ich kann es einfach nicht einfügen verdammt noch mal!

Wenn du Hilfe bei der Darstellung brauchst, nimmst du am besten via Kontaktknopf im grünen Balken unten mit dem Admin Kontakt auf. Ansonsten kann das untergehen.

Anm: Link zur Frage dort gleich angeben.

spielt das "t" eine große Rolle bei Matrizen? Was sagt mir das denn aus? 

^T für transponiert darfst du nicht unterschlagen.

Stimmt deine Kästchendarstellung für b) denn genau?

(Vektor x) mal (Vektor (i, - 2) ) ? als Skalarprodukt gemeint?

Links davon noch eine Matrix, die mit dem erwähnten Skalarprodukt multipliziert wird?

Hast du die TeX-Vorschau benutzt? https://www.matheretter.de/rechner/latex

Komme damit bei deinem b) auch nicht weiter.

aufgabe  b stimmt so, also wie ich das eingefügt habe. Ich habe es mit dem Formeleditor versucht ging nicht, auch mit dem $$ Zeichen ging das nicht.....deshalb habe ich das ganze dann als Bild gespeichert und dementsprechend eingefügt.....

bei b) stimmt jetzt alles...so wie es dargestellt ist soll ich rechnen, aber ich weiß nicht wie!!! ich habe keine ahnung :(((

die Aufgabenstellung ist so: Lösen Sie die folgenden Gleichungen wobei A eine Reelle 2*4 Matrix ist ( das wurde schon gerechnet) und  $$\xrightarrow { x } \quad \epsilon { \quad C }^{ 2 }$$ ist


und dann kommt halt die aufgabe b) so wie ich das eingefügt habe

Info zum TeX-Problem: Es wurde { pmatrix } geschrieben statt {pmatrix}. Der Mathjax-Parser hat damit leider ein Problem. Habe es oben korrigiert. Der Server wird zukünftig diese TeX-Anweisung selbst umwandeln und der Fehler nicht mehr auftreten.

Matheretter: Danke für den Hinweis. Die Umwandlung ist so gut gemacht, dass ich den Unterschied bei deinen " {pmatrix} geschrieben statt {pmatrix}" nur im HTML sehe. Du hast offenbar die Farbanweisung, font und alles Mögliche sonst noch entfernt.

Hi Lu. Es lag an den Leerzeichen bei den Klammern, diesmal nicht am HTML. Lg kai

Ein anderes Problem?

Stell deine Frage

Willkommen bei der Mathelounge! Stell deine Frage einfach und kostenlos

x
Made by a lovely community